LSAT and Law School Admissions Forum

Get expert LSAT preparation and law school admissions advice from PowerScore Test Preparation.

 Administrator
PowerScore Staff
  • PowerScore Staff
  • Posts: 8917
  • Joined: Feb 02, 2011
|
#59078
Please post your questions below!
User avatar
 msk21721
  • Posts: 1
  • Joined: Jun 14, 2021
|
#87918
For this question, I referenced the template set-up. I identified that the 1-3-2 template could satisfy the condition, with H under visitors with L. However, I couldn't figure out why D is incorrect if both F and K could be under R with the G-M block.
User avatar
 Ryan Twomey
PowerScore Staff
  • PowerScore Staff
  • Posts: 141
  • Joined: Mar 04, 2021
|
#88004
Hey Msk,

So for this question, I think you should do more than just identify the 1-3-2 template. You would want to see that H has to go into V with L as well. Then R has G, M, and F/K and then O has F/K .

I did this question by drawing two templates, one where H was in R and one where H was in V. The one where I placed H in R was eliminated because it did not work. The one where I placed H in V, did work with the placements above described.

Let me know if this makes sense. You are able to place everything in that template because of the not laws applying to F and K and the not laws applying to G and H.

Drawing the two possibilities of where H could go, and then eliminating one of those possibilities was very helpful for me in this question, and I wish you all of the luck in your studies.

Best,
Ryan
User avatar
 slynnnnn
  • Posts: 9
  • Joined: Jan 03, 2022
|
#93253
Hi PowerScore Team,

I got to the point where I know it has to be a 1-3-2 model, and that M&G have to go to [R]. Diagramed as below...

F/K/L M K/L
G H
F/K/L
__________________________________
O R V

Could you please further elaborate on how to get the inference that L has to go to [V]?

Thanks,
Lynn
User avatar
 slynnnnn
  • Posts: 9
  • Joined: Jan 03, 2022
|
#93254
...

The format seems off, but I had the setup as:
[O]: F/K/L
[R]: M, G, F/K/L
[V]: H, K/L
 Robert Carroll
PowerScore Staff
  • PowerScore Staff
  • Posts: 1787
  • Joined: Dec 06, 2013
|
#93384
slynnnnn,

The second rule says that K cannot go to V, so that reduces your dual K/L option in V to a single option - just L and H.

Robert Carroll

Get the most out of your LSAT Prep Plus subscription.

Analyze and track your performance with our Testing and Analytics Package.